GMAT Exam  >  GMAT Tests  >  Test: Logical Flaw - GMAT MCQ

Test: Logical Flaw - GMAT MCQ


Test Description

10 Questions MCQ Test - Test: Logical Flaw

Test: Logical Flaw for GMAT 2024 is part of GMAT preparation. The Test: Logical Flaw questions and answers have been prepared according to the GMAT exam syllabus.The Test: Logical Flaw MCQs are made for GMAT 2024 Exam. Find important definitions, questions, notes, meanings, examples, exercises, MCQs and online tests for Test: Logical Flaw below.
Solutions of Test: Logical Flaw questions in English are available as part of our course for GMAT & Test: Logical Flaw solutions in Hindi for GMAT course. Download more important topics, notes, lectures and mock test series for GMAT Exam by signing up for free. Attempt Test: Logical Flaw | 10 questions in 20 minutes | Mock test for GMAT preparation | Free important questions MCQ to study for GMAT Exam | Download free PDF with solutions
Test: Logical Flaw - Question 1

It is estimated that over 10 billion packs of cigarettes are smoked in the United States every year. An additional tax of 50 cents on every pack of cigarettes sold would achieve two desirable goals. First, the additional 5 billion dollars in revenue could be used to supplement the nation’s education budget and restore many programs that have been cut back or eliminated, such as vocational training and art education. In addition, the higher prices of buying cigarettes will encourage smokers to reduce the numbers of cigarettes that they smoke, which would help to improve the health of both the smokers and the bystanders who inhale their second-hand smoke.

Which of the following describes a flaw in the argument’s reasoning?

Detailed Solution for Test: Logical Flaw - Question 1

Let's go through each answer choice and explain why option (D) is the correct answer for the flaw in the reasoning.

(A) The author overlooks the possibility that education programs can be funded by means other than taxing cigarettes.
This answer choice suggests that the flaw lies in the assumption that cigarette taxes are the only possible means of funding education programs. However, the argument doesn't make this assumption. It simply proposes that the additional revenue from cigarette taxes could be used to supplement the education budget, without ruling out other funding sources. Therefore, (A) is not the correct answer.

(B) The author assumes that something that is sufficient to achieve one goal may be necessary to achieve another.
This answer choice suggests that the author assumes that the additional tax on cigarettes is both sufficient and necessary to achieve both the revenue goal and the health improvement goal. However, the argument doesn't explicitly make this assumption. It suggests that the tax would achieve both goals, but it doesn't claim that the tax is the only way to achieve these goals. Therefore, (B) is not the correct answer.

(C) The author ignores the beneficial results already achieved through public smoking bans to reduce the effects of second-hand smoke.
This answer choice suggests that the author overlooks the positive effects of public smoking bans on reducing second-hand smoke. However, the argument doesn't address or overlook the effects of smoking bans. It focuses on the potential benefits of higher cigarette prices due to the additional tax. Therefore, (C) is not the correct answer.

(D) The author fails to consider that success in achieving one of the proposal’s goals would negatively impact the success of the other.
This answer choice correctly identifies the flaw in the reasoning. The argument assumes that achieving one goal (revenue generation) would automatically lead to the success of the other goal (health improvement). However, the argument fails to consider that the success of one goal may negatively impact the success of the other. For example, if smokers reduce the number of cigarettes they smoke due to higher prices, the revenue generated from the tax may be lower than expected. Therefore, (D) is the correct answer.

(E) The author presumes a correlation between artificially low cigarette taxes and cutbacks in education programs.
This answer choice suggests that the author assumes a correlation between low cigarette taxes and cutbacks in education programs. However, the argument doesn't make this assumption. It proposes a solution to increase revenue for education programs but doesn't explicitly connect it to low cigarette taxes causing the cutbacks. Therefore, (E) is not the correct answer.

In conclusion, the flaw in the argument's reasoning is that the author fails to consider that success in achieving one of the proposal's goals (revenue generation) would negatively impact the success of the other goal (health improvement). This makes option (D) the correct answer.

Test: Logical Flaw - Question 2

John: There is no God because we cannot prove the existence of God
Mary: But inability to prove the existence of something doesn’t deny its existence

Mary attacks John’s argument by pointing out,

Detailed Solution for Test: Logical Flaw - Question 2

John's argument is as follows: "There is no God because we cannot prove the existence of God."

Mary attacks John's argument, stating, "But inability to prove the existence of something doesn't deny its existence."

Now, let's examine the answer options and their explanations:

(A) The invalid premise: This option suggests that the premise of John's argument is invalid. However, the premise itself is not addressed in the given dialogue, so this is not the correct answer.

(B) The erroneous conclusion: This option suggests that the conclusion drawn by John is incorrect. Mary is challenging John's conclusion that there is no God based on the inability to prove God's existence. Therefore, Mary is pointing out that John's conclusion is erroneous, making this the correct answer.

(C) The limited scope: This option refers to the argument's limited focus or narrow perspective. However, the dialogue does not suggest that either John or Mary's arguments have limited scope, so this is not the correct answer.

(D) A missing assumption: This option suggests that there is a missing assumption in John's argument. While it is possible that there may be a missing assumption, the dialogue does not provide evidence to support this, so this is not the correct answer.

(E) An inherent ambiguity: This option suggests that there is an inherent ambiguity in John's argument. However, the dialogue does not indicate any ambiguity, so this is not the correct answer.

Therefore, the correct answer is (B) the erroneous conclusion, as Mary is challenging John's conclusion that there is no God based on the inability to prove God's existence.

1 Crore+ students have signed up on EduRev. Have you? Download the App
Test: Logical Flaw - Question 3

That the policy of nuclear deterrence has worked thus far is unquestionable. Since the end of the Second World War, the very fact that there were nuclear armaments in existence has kept major powers from using nuclear weapons, for fear of starting a worldwide nuclear exchange that would make the land of the power initiating it uninhabitable. The proof is that a third world war between superpowers has not happened.

Which one of the following, if true, indicates a flaw in the argument?

Detailed Solution for Test: Logical Flaw - Question 3

Let's go through each answer choice and analyze how it indicates a potential flaw in the argument:

(A) Maintaining a high level of nuclear armaments represents a significant drain on a country's economy.
This answer choice doesn't necessarily indicate a flaw in the argument. While it suggests that maintaining nuclear armaments has an economic cost, it doesn't directly challenge the effectiveness of nuclear deterrence.

(B) From what has happened in the past, it is impossible to infer with certainty what will happen in the future, so an accident could still trigger a third world war between superpowers.
This answer choice points out the limitations of relying solely on past events to predict the future. It suggests that even though a third world war hasn't occurred yet, it doesn't guarantee that it won't happen in the future due to unforeseen accidents or events. This undermines the argument's assertion that nuclear deterrence has worked without question.

(C) Continuing to produce nuclear weapons beyond the minimum needed for deterrence increases the likelihood of a nuclear accident.
This answer choice suggests that producing excessive nuclear weapons beyond what is necessary for deterrence could lead to a higher probability of a nuclear accident. If nuclear accidents were to occur, it would challenge the argument's assumption that the existence of nuclear weapons alone has been sufficient to prevent the use of nuclear weapons by major powers.

(D) The major powers have engaged in many smaller-scale military operations since the end of the Second World War, while refraining from a nuclear confrontation.
This answer choice presents evidence that major powers have participated in smaller-scale military operations without resorting to nuclear confrontation. This challenges the argument's assertion that the existence of nuclear weapons is the sole reason for major powers refraining from nuclear conflict.

(E) It cannot be known whether it was nuclear deterrence that worked, or some other factor, such as a recognition of the economic value of remaining at peace.
This answer choice raises the possibility that factors other than nuclear deterrence, such as economic considerations, may have played a role in preventing a third world war between superpowers. It suggests that the argument's assumption attributing the absence of a third world war solely to nuclear deterrence is flawed.

Given the above analysis, the answer that indicates a flaw in the argument is (E). It suggests that the effectiveness of nuclear deterrence cannot be conclusively determined, as other factors may have influenced the avoidance of a third world war.

Test: Logical Flaw - Question 4

Mayor: The law prohibiting pedestrians from crossing against red lights serves no useful purpose. After all, in order to serve a useful purpose, a law must deter the kind of behavior it prohibits. But pedestrians who invariably violate this law are clearly not dissuaded by it; and those who comply with the law do not need it, since they would never cross against red light even if there were no law prohibiting pedestrians from crossing against red lights.

The mayor’s argument is flawed because it

Detailed Solution for Test: Logical Flaw - Question 4

The mayor's argument can be summarized as follows:

  • The law prohibiting pedestrians from crossing against red lights serves no useful purpose.
  • Pedestrians who violate this law are not dissuaded by it.
  • Pedestrians who comply with the law do not need it.

The flaw in the mayor's argument lies in the assumption that the law does not dissuade people who sometimes but not always cross against red lights. The argument assumes that people either invariably violate the law or never violate it. However, there may be individuals who occasionally cross against red lights but are dissuaded by the law from doing so more frequently. The mayor fails to consider this possibility, which weakens the argument.

Now, let's analyze each answer choice:

(A) takes for granted that most automobile drivers will obey the law that prohibits them from driving through red lights.
This answer choice introduces a separate issue concerning automobile drivers, which is not directly relevant to the argument. It does not address the flaw in the mayor's argument, so it can be eliminated.

(B) uses the word "law" in one sense in the premises and in another sense in the conclusion.
This answer choice suggests that there is a discrepancy in how the word "law" is used, but there is no evidence of such a discrepancy in the argument. The argument consistently refers to the law prohibiting pedestrians from crossing against red lights. Therefore, (B) can be eliminated.

(C) ignores the possibility that a law might not serve a useful purpose even if it does deter the kind of behavior it prohibits.
This answer choice introduces a new perspective, suggesting that a law might not serve a useful purpose even if it does deter the prohibited behavior. However, this is not the flaw in the mayor's argument. The mayor's argument focuses on the efficacy of the law in deterring behavior, not on whether a law can serve a useful purpose without deterring behavior. Thus, (C) is not the correct answer.

(D) fails to consider whether the law ever dissuades people who sometimes but not always cross against red lights.
This answer choice accurately identifies the flaw in the mayor's argument. The argument assumes that people either invariably violate the law or never violate it, without considering the possibility of individuals who occasionally cross against red lights but are dissuaded by the law from doing so more frequently. Therefore, (D) is the correct answer.

(E) provides no evidence that crossing against red lights is more dangerous than crossing on green lights.
This answer choice introduces a new consideration regarding the relative danger of crossing against red lights versus crossing on green lights. However, the argument does not rely on or require evidence about the comparative danger of these actions. Thus, (E) can be eliminated.

In conclusion, the correct answer is (D) because it accurately identifies the flaw in the mayor's argument. The mayor fails to consider whether the law dissuades people who sometimes but not always cross against red lights.

Test: Logical Flaw - Question 5

Reviewer: The Plain Truth, a biography of the philosopher Socrates, states that Socrates was both very wise and very poor. The book must be incorrect, because a man possessing such wisdom would surely find a way to avoid poverty.

Which of the following best shows the most significant flaw in the reviewer's argument?

Detailed Solution for Test: Logical Flaw - Question 5

The question asks us to identify the most significant flaw in the reviewer's argument that Socrates couldn't have been both wise and poor. We need to find the answer choice that weakens or undermines this argument.

Let's analyze each answer choice:

(A) Every individual who is wise desires to be a philosopher.
This answer choice introduces a new statement that is not directly related to the flaw in the reviewer's argument. It doesn't address the connection between wisdom and poverty. Therefore, it is not the correct answer.

(B) Socrates would not have been executed by the city of Athens if he had not been influential.
This answer choice discusses Socrates' execution and its connection to his influence. While it may suggest that Socrates was indeed influential, it doesn't directly address the flaw in the reviewer's argument regarding the relationship between wisdom and poverty. Hence, it is not the correct answer.

(C) Conceptions of wealth vary over time, and what is now considered poor would have been very wealthy in Socrates' time.
This answer choice challenges the assumption that being poor in Socrates' time meant the same as being poor today. It suggests that the definition and standards of wealth can vary across different periods. This undermines the reviewer's argument by highlighting that our understanding of poverty may differ from that of Socrates' time. This answer choice weakens the reviewer's argument and could be a potential flaw. However, it is not as significant as the correct answer.

(D) Many people who are considered wise are too preoccupied with pursuing wisdom to care about their economic status.
This answer choice directly addresses the flaw in the reviewer's argument by suggesting that wise individuals may prioritize the pursuit of wisdom over their economic status. It implies that Socrates, being wise, may have been more focused on wisdom rather than finding a way to avoid poverty. This weakens the assumption that a wise person would necessarily find a way to avoid poverty. Therefore, (D) is the correct answer.

(E) Although Socrates was not wealthy, he had many wealthy friends who willingly took care of him.
This answer choice suggests that Socrates had wealthy friends who supported him, despite his own lack of wealth. While this information challenges the reviewer's assumption that Socrates couldn't have been both wise and poor, it doesn't directly address the flaw in the argument. It focuses more on Socrates' social network rather than the relationship between wisdom and poverty. Therefore, it is not the correct answer.

In summary, the correct answer is (D) because it directly addresses the flaw in the reviewer's argument by suggesting that wise individuals may be preoccupied with pursuing wisdom rather than their economic status.

Test: Logical Flaw - Question 6

Although City International School has provided qualified teachers and a revamped curriculum for the last three years, the social science and geography scores for grade X students have failed to reach the expected levels. Therefore, the parents have recommended to the principal that the text-heavy subjects such as social science and geography be taught in their native language instead of in English.

The recommendation of the parents is flawed because they failed to consider whether __________.

Detailed Solution for Test: Logical Flaw - Question 6

Let's analyze each answer choice and determine the correct option:

(A) the undesirable outcome is an exception: This answer choice suggests that the undesirable outcome is an isolated incident, implying that it is not a recurring issue. However, the parents' recommendation is still flawed regardless of whether the outcome is exceptional or not. Hence, this choice does not directly address the flaw in the parents' recommendation.

(B) only one cause led to the undesirable outcome: This answer choice correctly identifies the flaw in the parents' recommendation. The recommendation assumes that the issue with the social science and geography scores is solely due to the subjects being taught in English. However, it overlooks the possibility that other factors may contribute to the students' poor performance. Therefore, this choice highlights the parents' failure to consider alternative causes, making it the correct answer.

(C) the irrelevant recommendation will address the concern: This answer choice suggests that even though the parents' recommendation may not directly address the concern, it could still have some beneficial impact. However, the flaw in the recommendation lies in its failure to consider alternative causes, rather than whether the recommendation is relevant or not. Hence, this choice does not directly address the flaw in the parents' recommendation.

(D) only one cause led to many undesirable outcomes: This answer choice is similar to choice (B) but introduces the notion of multiple undesirable outcomes. However, the parents' recommendation focuses specifically on the social science and geography scores, so this choice goes beyond the scope of the recommendation. Therefore, it does not directly address the flaw in the parents' recommendation.

(E) many causes led to many undesirable outcomes: This answer choice broadens the scope beyond the social science and geography scores to include multiple causes and outcomes. However, the parents' recommendation is specifically related to the subjects being taught in English and does not consider other causes. Therefore, this choice goes beyond the flaw in the parents' recommendation and is not directly relevant.

In conclusion, option (B) correctly identifies the flaw in the parents' recommendation and provides a direct explanation of why their recommendation is flawed.

Test: Logical Flaw - Question 7

Any announcement authorized by the head of the department is important. However, announcements are sometimes issued, without authorization, by people other than the head of the department, so some announcements will inevitably turn out not to be important.

The reasoning is flawed because the argument

Detailed Solution for Test: Logical Flaw - Question 7

Let's analyze each answer choice and see which one correctly identifies the flaw in the argument:

(A) does not specify exactly which communications are to be classified as announcements.
This answer choice points out that the argument lacks clarity in defining what qualifies as an announcement. While this may be a potential flaw, it does not address the main issue in the argument.

(B) overlooks the possibility that people other than the head of the department have the authority to authorize announcements.
This answer choice suggests that there could be individuals other than the head of the department who are authorized to issue announcements. While this could be a flaw, it is not the central flaw in the argument.

(C) leaves open the possibility that the head of the department never, in fact, authorizes any announcements.
This answer choice highlights the possibility that the head of the department may never actually authorize any announcements. While this could be a flaw, it is not the main flaw in the argument.

(D) assumes without warrant that just because satisfying a given condition is enough to ensure an announcement's importance, satisfying that condition is necessary for its importance.
This answer choice correctly identifies the flaw in the argument. The argument assumes that any announcement authorized by the head of the department is important, without considering the possibility that there could be other factors influencing the importance of an announcement. It points out that while satisfying the condition of authorization by the head of the department may be enough to ensure an announcement's importance, it is not necessary for its importance.

(E) fails to distinguish between the importance of the position someone holds and the importance of what that person may actually be announcing on a particular occasion.
This answer choice highlights the distinction between the importance of the position held by someone and the importance of the specific announcement they are making. While this could be a flaw, it is not the central flaw in the argument.

Therefore, the correct answer is (D) because it accurately identifies the flaw in the reasoning presented in the argument.

Test: Logical Flaw - Question 8

A study of the dietary habits of a group of people who had recently developed cancer and a group without cancer found that during the previous five years the diets of"
"the two groups' members closely matched each other in the amount of yogurt they contained. Yogurt contains galactose, which is processed in the body by an enzyme. In the people with cancer the levels of this enzyme were too low to process the galactose in the yogurt they were consuming. It can be concluded that galactose in amounts exceeding the body's ability to process it is carcinogenic.

Of the following, which one constitutes the strongest objection to the reasoning in the argument?

Detailed Solution for Test: Logical Flaw - Question 8

Let's go through each answer choice and evaluate its impact on the argument:

(A) The argument fails to consider whether the dietary habits of everyone in the two groups were the same in all other respects.

This objection points out that the argument does not account for other dietary factors that could potentially influence the development of cancer. It suggests that there might be other dietary habits or components that differ between the two groups, which could be the actual cause of cancer. This objection weakens the argument, as it raises doubts about the conclusion drawn solely based on the similarity in yogurt consumption. Therefore, (A) is a strong objection.

(B) The argument neglects to recommend that people with low levels of the enzyme avoid eating yogurt.

This objection focuses on the practical implications of the argument. While it suggests a possible course of action for individuals with low levels of the enzyme, it does not directly weaken the argument's logic. It does not challenge the conclusion or the causal relationship between galactose and cancer. Therefore, (B) is a weak objection.

(C) The argument focuses on only one substance that can increase the risk of cancer when it is well known that there are many such substances.

This objection highlights the limited scope of the argument, which solely focuses on galactose and fails to consider other substances that may contribute to cancer development. However, this objection does not directly challenge the argument's reasoning or the conclusion drawn from the study. It merely points out that there are other potentially relevant factors to consider. Therefore, (C) is a weak objection.

(D) The argument overlooks the possibility that cancer causes low levels of the enzyme.

This objection suggests an alternative causal relationship between cancer and the enzyme levels. It points out that the low levels of the enzyme could be a result of cancer rather than the cause of cancer. If cancer itself leads to low levels of the enzyme, then the argument's conclusion about galactose being carcinogenic would be undermined. Therefore, (D) is a strong objection.

(E) The argument does not specify whether any member of either group lacked the enzyme entirely.

This objection raises a question about the enzyme levels in both groups but does not directly challenge the argument's reasoning or conclusion. While it may introduce some uncertainty, it does not provide significant evidence against the conclusion. Therefore, (E) is a weak objection.

In summary, the strongest objection to the reasoning in the argument is (D) because it presents an alternative causal relationship that could undermine the conclusion.

Test: Logical Flaw - Question 9

Reva: Using extraneous incentives to get teenagers to change their attitude toward school and schoolwork won’t work. Take the program in West Virginia, for instance, where they tried to reduce their dropout rate by revoking the driving licenses of kids who left school. The program failed miserably.

Anne: It’s true that the West Virginia program failed, but many schools have devised incentive programs that have been very successful in improving attendance and reducing discipline problems.

According to Anne, the weak point in Reva’s claim is that it

Detailed Solution for Test: Logical Flaw - Question 9

Let's go through each option and analyze them one by one:

(A) fails to consider the possibility that the majority of potential dropouts in West Virginia do not have driving licenses
This option suggests that the West Virginia program may have failed because a significant portion of potential dropouts did not have driving licenses to begin with. However, this argument does not address the effectiveness of extraneous incentives in general, so it is not directly relevant to the weak point in Reva's claim.

(B) doesn't provide any exact figures for the dropout rate in West Virginia before and during the program
This option suggests that without specific dropout rate figures, it is difficult to determine the success or failure of the program. While the lack of precise data may weaken the argument, it does not directly address the effectiveness of extraneous incentives or Reva's claim. Therefore, it is not the weak point in Reva's claim.

(C) ignores a substantial body of evidence showing that parents and employers have been using extrinsic incentives with positive results for years
This option points out that there is evidence supporting the use of extrinsic incentives by parents and employers, which implies that such incentives can be effective. It challenges Reva's claim by highlighting the existence of positive results achieved through extraneous incentives. This is a strong response that undermines Reva's claim.

(D) assumes that a positive incentive—a prize or a reward—will be no more effective than a negative incentive, like the revoking of a driving license
This option suggests that Reva assumes that positive and negative incentives have similar effectiveness, without considering that positive incentives may be more effective. However, Reva's claim is focused on the ineffectiveness of extraneous incentives in general, rather than comparing positive and negative incentives. Therefore, this option does not address the weak point in Reva's claim.

(E) is based on a single example, the incentive program in West Virginia, which may not be typical
This option points out that Reva's claim is based on a single example, which is the West Virginia program. By highlighting the lack of generalizability, it weakens Reva's claim as it suggests that the failure of one program does not necessarily invalidate the effectiveness of extraneous incentives in all cases. This option directly addresses the weak point in Reva's claim.

Therefore, the correct answer is (E) is based on a single example, the incentive program in West Virginia, which may not be typical.

Test: Logical Flaw - Question 10

Some employees have raised doubts about the manager’s impartiality in awarding performance ratings. He was discovered to be biased towards female employees than towards male employees. But the record shows that he has given a rating of 4 or higher (upon 5) to 80% of the male employees under him while giving such ratings to only 65% of the female employees. This clearly shows that the manager has not discriminated against male employees during performance appraisal.

The argument above is flawed in that it ignores the possibility that

Detailed Solution for Test: Logical Flaw - Question 10

The argument states that some employees have raised doubts about the manager's impartiality in awarding performance ratings, specifically suggesting that he is biased towards female employees. The argument then presents some statistics to refute this claim, stating that the manager has given a rating of 4 or higher to 80% of the male employees and only 65% of the female employees. The conclusion drawn is that the manager has not discriminated against male employees.

To identify the flaw in the argument, we need to find an option that presents a possible alternative explanation or assumption that undermines the conclusion. Let's analyze the options provided:

(A) A large number of the employees whom the manager evaluated were male.
This option suggests that there were more male employees than female employees evaluated by the manager. However, this information doesn't directly address the manager's impartiality or bias.

(B) Many managers find it difficult to be objective when awarding ratings to employees since they always have a personal bias/favorites.
This option suggests a general tendency among managers, but it doesn't specifically address the situation of the manager in question or the gender bias observed. Therefore, it is not directly relevant to the argument.

(C) The manager is more biased against male employees who complained to HR department than the ones who did not.
This option introduces a new factor of bias against male employees who complained to HR. While it addresses potential bias, it doesn't directly address the observed bias towards female employees or explain the discrepancy in performance ratings between male and female employees.

(D) The partiality against male employees that the manager is accused of has been of male employees who did not get good ratings last year.
This option suggests that the bias against male employees is limited to those who did not receive good ratings the previous year. However, this information doesn't directly address the observed bias towards female employees or explain why a higher percentage of male employees received good ratings compared to female employees.

(E) A greater percent of male employees deserved to get 4+ ratings, while a smaller percent of female employees should have got such ratings, than the final numbers.
This option provides an alternative explanation for the observed bias. It suggests that a higher percentage of male employees deserved high ratings, while a lower percentage of female employees deserved them. This alternative explanation undermines the argument's conclusion that the manager's ratings were not biased against female employees.

Therefore, the correct answer is (E) because it presents a plausible alternative explanation for the observed bias in the manager's ratings.

Information about Test: Logical Flaw Page
In this test you can find the Exam questions for Test: Logical Flaw solved & explained in the simplest way possible. Besides giving Questions and answers for Test: Logical Flaw, EduRev gives you an ample number of Online tests for practice

Top Courses for GMAT

Download as PDF

Top Courses for GMAT